LSAT and Law School Admissions Forum

Get expert LSAT preparation and law school admissions advice from PowerScore Test Preparation.

 Administrator
PowerScore Staff
  • PowerScore Staff
  • Posts: 8917
  • Joined: Feb 02, 2011
|
#28606
Complete Question Explanation
(The complete setup for this game can be found here: lsat/viewtopic.php?t=11942)

The correct answer choice is (C)

When addressing this question stem, first convert the Must Be False statement into its true equivalent, Cannot Be True. Thus, the one correct answer Cannot Be True, and the four incorrect answers Could Be True.

The local condition in the question stem, L :longline: K, is not easy to handle. The rule adds another layer of complexity to an already complex diagram. The difficulty in the L :longline: K relationship comes from the fact that L and K are already “floating” because of their relationships with H and O, respectively. To show that the two floating variables have a relationship, and most importantly, to understand the implications of that relationship, is challenging. The diagram would appear as:
Dec 06_M12 game #4_cr_game#4_#22_diagram_1.png
This diagram is deceptive because it makes it appear as though L must be delivered relatively early, and that K must be delivered relatively late. This is not true: L can be delivered as late as sixth (followed by K and O), and K can be delivered as early as third (preceded by H and L). When answering this question—which may be the toughest of the section—be very careful and deliberate.

Answer choice (A): This answer choice is incorrect because N could be the second parcel delivered. The following hypothetical shows how: J-N-H-L-K-M-G-O.

Answer choice (B): This answer choice is incorrect because L could be the third parcel delivered. The following hypothetical shows how: H-N-L-J-M-G-K-O.

Answer choice (C): This is the correct answer choice. Because H must all be delivered earlier than M, G, K, L, and K, the latest that H can be delivered is third (after J and N).

Answer choice (D): This answer choice is incorrect because K could be the fifth parcel delivered. The following hypothetical shows how: N-J-H-L-K-M-G-O.

Answer choice (E): This answer choice is incorrect because M could be the sixth parcel delivered. The following hypothetical shows how: J-N-H-L-K-M-G-O.
You do not have the required permissions to view the files attached to this post.

Get the most out of your LSAT Prep Plus subscription.

Analyze and track your performance with our Testing and Analytics Package.